Wykazać równość

Własności funkcji trygonometrycznych i cyklometrycznych. Tożsamości. RÓWNANIA I NIERÓWNOŚCI.
tranto
Użytkownik
Użytkownik
Posty: 64
Rejestracja: 3 paź 2009, o 20:20
Płeć: Kobieta
Podziękował: 12 razy

Wykazać równość

Post autor: tranto »

Udowodnij, że jeżeli \(\displaystyle{ \frac{sin^4 \alpha}{a} + \frac{cos^4 \alpha}{b} = \frac{1}{a+b}}\), to \(\displaystyle{ \frac{sin^8 \alpha}{a^3} + \frac{cos^8 \alpha}{b^3} = \frac{1}{a+b}}\).

Wydaje mi się, że znalazłam dowód, proszę o sprawdzenie.

Zakładamy, że \(\displaystyle{ a, b, (a+b) \neq 0}\).
Wprowadzamy zmienne pomocznicze x, y:
\(\displaystyle{ \begin{cases} x = sin^2 \alpha\\ y=cos^2 \alpha \end{cases}}\)
Oczywiście, zachodzi \(\displaystyle{ x+y=1}\). Wtedy mamy:
\(\displaystyle{ \frac{sin^4 \alpha}{a} + \frac{cos^4 \alpha}{b} = \frac{1}{a+b} \Leftrightarrow}\)
\(\displaystyle{ \frac{x^2}{a} + \frac{(1-x)^2}{b} = \frac{1}{a+b} \Leftrightarrow}\)
\(\displaystyle{ x^2 + \frac{b}{a} x^2 + (1-x)^2 + \frac{a}{b} (1-x)^2 = 1}\)

Wprowadzamy zmienną pomocniczą \(\displaystyle{ k= \frac{a}{b}}\), rozwiązujemy to równanie względem x i otrzymujemy: \(\displaystyle{ x = \frac{k}{k+1} \Leftrightarrow x=\frac{a}{a+b}}\).
Stąd (ponieważ \(\displaystyle{ y = 1 - x}\)):
\(\displaystyle{ y= \frac{b}{a+b}}\).

W takim razie mamy:
\(\displaystyle{ \frac{sin^8 \alpha}{a^3} + \frac{cos^8 \alpha}{b^3} = \left(\frac{x}{a}\right)^3 +\left(\frac{y}{b} \right)^3 = \frac{2}{(a+b)^3}}\)

Wydaje mi się, że wkradł się błąd drukarski, ponieważ
\(\displaystyle{ \frac{2}{(a+b)^3} = \frac{1}{a+b} \Leftrightarrow 2(a+b) = (a+b)^3 \Leftrightarrow (a+b)^2=2 \Leftrightarrow (a+b) = \pm \sqrt2}\).

Dla danych \(\displaystyle{ a = b = sin^2 \alpha = cos^2 \alpha = \frac{1}{2}}\) teza nie zachodzi.
(Istnienie kąta \(\displaystyle{ \alpha}\) spełniającego ten warunek jest zapewnione przez odpowiednie twierdzenie).
Użytkownik
Użytkownik
Posty: 9833
Rejestracja: 18 gru 2007, o 03:54
Płeć: Mężczyzna
Lokalizacja: Bydgoszcz
Podziękował: 90 razy
Pomógł: 2632 razy

Wykazać równość

Post autor: »

tranto pisze:\(\displaystyle{ x^2 + \frac{b}{a} x^2 + (1-x)^2 + \frac{a}{b} (1-x)^2 = 1}\)
Wprowadzamy zmienną pomocniczą \(\displaystyle{ k= \frac{a}{b}}\), rozwiązujemy to równanie względem x i otrzymujemy: \(\displaystyle{ x = \frac{k}{k+1} \Leftrightarrow x=\frac{a}{a+b}}\).
Na pewno?
tranto pisze:Dla danych \(\displaystyle{ a = b = sin^2 \alpha = cos^2 \alpha = \frac{1}{2}}\) teza nie zachodzi.
Akurat dla tych zachodzi, ale dla \(\displaystyle{ a=b=1, \sin^2\alpha= \cos^2\alpha = \frac{1}{2}}\) istotnie nie. I to wystarczy, kontrprzykład obala tezę, nic więcej nie trzeba pisać.

Q.
tranto
Użytkownik
Użytkownik
Posty: 64
Rejestracja: 3 paź 2009, o 20:20
Płeć: Kobieta
Podziękował: 12 razy

Wykazać równość

Post autor: tranto »

Qń pisze:Na pewno?
Sprawdzam obliczenia i nie widzę tam błędu. Problem jest chyba inny:
tranto pisze: \(\displaystyle{ x = sin^2 \alpha}\)
tranto pisze: \(\displaystyle{ \frac{sin^8 \alpha}{a^3} + \frac{cos^8 \alpha}{b^3} = \left(\frac{x}{a}\right)^3 +\left(\frac{y}{b} \right)^3}\)

Stąd wychodziło mi, że teza nie zachodzi dla danych \(\displaystyle{ a = b = sin^2 \alpha = cos^2 \alpha = \frac{1}{2}}\).
W takim razie byłoby:
\(\displaystyle{ \frac{sin^8 \alpha}{a^3} + \frac{cos^8 \alpha}{b^3} = \left(\frac{x}{a}\right)^3 x + \left(\frac{y}{b}\right)^3 y = \left(\frac{1}{a+b}\right)^3 x + \left(\frac{1}{a+b}\right)^3 y =}\)
\(\displaystyle{ = \left(\frac{1}{a+b}\right)^3 \cdot (x+y)= \left(\frac{1}{a+b}\right)^3}\)
Użytkownik
Użytkownik
Posty: 9833
Rejestracja: 18 gru 2007, o 03:54
Płeć: Mężczyzna
Lokalizacja: Bydgoszcz
Podziękował: 90 razy
Pomógł: 2632 razy

Wykazać równość

Post autor: »

A, rzeczywiście, to mi się pozajączkowało.

W każdym razie powtórzę rzecz najważniejszą: w rozwiązaniu wystarczy podać kontrprzykład, wszystkie rachunki prowadzące do znalezienia kontrprzykładu są zbędne (w sensie: nie są częścią rozwiązania).

Q.
ODPOWIEDZ